3
$\begingroup$

A bilinear argument of the Fourier restriction (not bilinear Fourier restriction) can be described follows:

enter image description here

This excerpt is from the book of C. Demeter, Fourier restriction, decoupling, applications. Using the coarea formula, we can reformulate this argument in other $\mathbb R^n$, not just in $\mathbb R^2$:

WLOG let $n=3$, using $E$ as the extension operator of the parabola, $$ \|Ef\|_3^2 = \| (Ef) ^2\|_{3/2}, $$ $T\colon \mathbb R^4 \to\mathbb R^3$ is $$ T(t,s) = (t+s, |t|^2+|s|^2) , $$ then by the coarea formula ($\mathrm J_T$ means the Jacobian of $T$) $$ \begin{aligned} (Ef)^2(x) & = \int_{(t,s)\in\mathbb R^4,t\neq s} f(t)f(s) e\bigl(\,\langle\bar x, (t+s)\rangle+ x_3(|t|^2+|s|^2)\,\bigr) \,d(t,s)\\ & = \int_{\mathbb R^3}\biggl[\int_{t\neq s, T(t,s)=\xi} \frac{f(t)f(s)}{\mathrm{J}_T(t,s)} \,d \mathcal H^1(t,s) \biggr]e(\langle x,\xi\rangle)\,d\xi. \end{aligned} $$ Hence by Hausdorff-Young, $\|(Ef)^2\|_{3/2}\leq \|F\|_3$, where $$ F(\xi) = \int_{t\neq s, T(t,s)=\xi} \frac{f(t)f(s)}{\mathrm{J}_T(t,s)} \,d \mathcal H^1(t,s). $$ By the rank theorem, the level set of $T$ is the union of some curves; since $|(t,s)|\leq1$, the $\mathcal H^1$-measure of the level set is uniformly bounded, hence $$ |F(\xi)| \lesssim \biggl( \int_{t\neq s, T(t,s)=\xi} \Bigl| \frac{f(t)f(s)}{\mathrm{J}_T(t,s)} \Bigr|^3 \,d \mathcal H^1(t,s) \biggr)^{1/3}, $$ and the $L^3$ norm of $F$ is $$ \|F\|_3^3\lesssim \int_{\mathbb R^3}\biggl[\int_{t\neq s, T(t,s)=\xi} \biggl|\frac{f(t)f(s)}{\mathrm{J}_T(t,s)}\biggr|^3 \,d \mathcal H^1(t,s) \biggr]\,d\xi = \int_{\mathbb R^4} \frac{|f(t)f(s)|^3}{|\mathrm{J}_T(t,s)|^2}\,d(t,s). $$ By Hardy-Littlewood-Sobolev estimate (for convenience, we assume that the Hardy-Littlewood-Sobolev inequality holds at the endpoints), and the fact that $$ \mathrm{J}_T(t,s) = c |t-s|, $$ we can deduce that $$ \|F\|_3^3\lesssim \| \,|f|^3\,\|_1 \Bigl\| |f|^3 *\frac1{|\cdot|^2}\Bigr\|_\infty \lesssim \|f\|_3^3 \|\,|f|^3\,\|_3 \lesssim \|f\|_\infty^6, $$ which implies the restriction conjecture. Therefore, there must be some mistakes in it, but I can't figure out where the error lies.

$\endgroup$
2
  • $\begingroup$ @LVFToENlCx In 4-dim, the convolution kernel is locally integratable, hence if $\|f\|_\infty \leq 1$, then $\|F\|_3<\infty$, which can show that $\|F\|_3^3 \lesssim \|f\|_\infty^2$. $\endgroup$ Commented Sep 21 at 17:15
  • $\begingroup$ @LVFToENlCx The $DT(t,s)$ is $\left( \begin{array}{cccc} 1 & 0 & 1 & 0 \\ 0 & 1 & 0 & 1 \\ 2 t_1 & 2 t_2 & 2 s_1 & 2 s_2 \\ \end{array} \right)$, hence $(DT)(DT)^\top$ is $\left( \begin{array}{ccc} 2 & 0 & 2 \left(s_1+t_1\right) \\ 0 & 2 & 2 \left(s_2+t_2\right) \\ 2 \left(s_1+t_1\right) & 2 \left(s_2+t_2\right) & 4 \left(s_1^2+s_2^2+t_1^2+t_2 ^2\right) \\ \end{array} \right)$, whose det is $8 \left(\left(s_1-t_1\right){ }^2+\left(s_2-t_2\right){}^ 2\right) = C|t-s|^2$. $\endgroup$ Commented Oct 7 at 14:49

0

You must log in to answer this question.

Start asking to get answers

Find the answer to your question by asking.

Ask question

Explore related questions

See similar questions with these tags.